JEE Main & Advanced Mathematics Sets Question Bank Numerical Value Type Questions - Sets

  • question_answer
    Given \[n(U)=20,\] \[n(A)=12,\] \[n(B)=9,\]\[n(A\cap B)=4,\] where U is the universal set, A and B are subsets of U, then \[n({{(A\cup B)}^{c}})=\]

    Answer:

    3


You need to login to perform this action.
You will be redirected in 3 sec spinner